PLEASE HELP ME ITS ALGEBRA THANK YOUUUU

PLEASE HELP ME ITS ALGEBRA THANK YOUUUU

Answers

Answer 1

88km

26/13 = x/44

44*26 = 13x

1144 = 13x

x= 88

Answer 2

Answer:

88

Step-by-step explanation:

[tex] \frac{26}{13} = \frac{x}{44} [/tex]

reduce the first ratio

[tex] \frac{2}{1} = \frac{x}{44} [/tex]

cross multiply and divide

[tex]x = 88[/tex]


Related Questions

Kaitlin drove 325 miles in 5 hours. At the same rate, how long would it take her to drive 455 miles?

Answers

Answer:

7 hours

Step-by-step explanation:

take the original distance traveled to find the speed traveled and take the new distance to travel devided by the speed 325÷5=65 455÷65=7

Is the problem situation a function? Explain your answer.

Answers

Answer:

where is the problem at

Step-by-step explanation:

como se saca la raíz cuadrada?

Answers

Answer:

La raíz cuadrada de cualquier número se puede expresar usando la fórmula: √y = y½.

A down payment of $195 is made on a television that costs $1300. What percent of the
cost is the down payment?

Answers

6.66666667% I hope that is helpful to you

Answer:

15%

Step-by-step explanation:

Add 195 to 1300, which gives you, 1495.

Use the formula for percent change:  Difference = % * original price.

So what is the difference between 1495 and 1300? It’s 195.

Now we have 195 as the difference.

Do we know the original price of the TV? Yes we do. What is it? Its 1300 because the question states the TV costs $1300.

Now we have the equation 195 = % * 1300

We don’t know the percentage, right? So now we have to solve for %. To do that, you have to do 195 divided by 1300, which gives you 0.15. Lastly, you have to turn 0.15 into a percentage, to do that, multiply 0.15 by 100, and it should give you 15%.

Therefore, 15% is the down payment. Hope this helps!

List all the 3-digit numbers that fit
these clues.
• The hundreds digit is greater
than 7.
• The tens digit is less than 2.
• The ones digit is the same as the
hundreds digit.
So, the numbers that fit the clues are 191, 193, 195, 197, 199.

Answers

i’m sorry, i don’t quite understand the question. do you have a photo of it by any chance?

divide the following mixed fraction
2 7/10 ÷ 3 1/4

Answers

Answer:

54/155

Step-by-step explanation:

do the reciprocal of 31/4then multiply it with27/10

Answer:

54/155

Step-by-step explanation:

hope it works out for you

If 8 pizzas cost $72,how much will 15 pizzas cost

Answers

Answer: $135

Explanation: To find how much 15 pizzas will cost, first we have to find out how much just one pizza will cost. To do this, you have to divide both the number of pizzas (8) and also the price for those 8 pizzas ($72) by the number of pizzas (8 again). When you do this you get the price of one pizza. Then you multiply the price of one pizza by the amount of pizzas you actually need to find the value for (15). This is a lot to explain but don't lose hope! Let me show you step by step and it'll make more sense.

Step 1) Divide both the amount of pizzas and the cost of that many pizzas

8 ÷ 8 = 1

72 ÷ 8 = 9

This means that one pizza costs 9 dollars.

Step 2) Multiply the cost of one pizza by 15 (the amt of pizzas you need to find the cost for)

15 × 9 = 135

135 is the cost for 15 pizzas. I hope this helps you!

So sense the total of 8 pizzas equal 72 we need to know how much the cost is for 1 pizza.

In that case we will do 72 divided by 8. And that equals 9.

Now that we know the cost for 1 pizza each is (9),

we do 9x15 to get the total cost of 15 pizzas and that is $135.

Answer:$135

simple❤️

Use the cards below to create a list of steps, in order, that will solve the following equation.
3(x+6)2 = 75
Solution steps:
Add 6 to both sides
Divide both sides by 3
1
Divide both sides by
3
Subtract 6 from both sides
ay
23
Square both sides
Take the square root of both sides

Answers

1. Divide both sides by 3
2. Take the square root of both sides
3. Subtract 6 from both sides


Graph system of inequalities
Y>|2/3x-3
Y>|-4/3x+3


Answers

The Graphs are in attachment !

1st graph is for equation :

[tex]y > \dfrac{2}{ 3}x - 3[/tex]

2nd graph is for equation :

[tex]y > - \dfrac{4}{3} x + 3[/tex]

Given system of inequalities:-

[tex]y = \frac{2}{3}x - 3[/tex]

and

[tex]y = \frac{ - 4}{3}x + 3[/tex]

So, the graph for the given system of inequalities is attached along with this answer.

Please refer the attachment for the graph.

✍️ By Benjemin ☺️

5(t-9) +12t=20

3(6m-17),m=5

Answers

Answers: 67 and 39

Solving:
5(t - 9) + 12
5(20 - 9) + 12
100 - 45 + 12
55 + 12
67

3(6m - 17)
3(6*5 - 17)
3(30 - 17)
90 - 51
39

Click this if you are a savage

Answers

Answer:

i  am

Step-by-step explanation:

pro!

uhfy1gujhqfvwtfu

Given the function y = (x - 2)2 graphed below, which restriction of the domain will result in
the inverse being a function?

VIEW IMAGE FOR GRAPH!

A: y = (x - 2)^2 , x ≥ 1
B: y = (x - 2)^2 , x ≥ 2
C: y = (x - 2)^2 , x ≥ 0
D: y = (x - 2)^2 , x ≥ -2

Answers

The answer is B: Y=(x-2)^2, x> 2

The restriction of the domain of the function will be y = (x - 2)², x ≥ 2. Then the correct option is B.

What is a function?

A statement, principle, or policy that creates the link between two variables is known as a function. Functions are found all across mathematics and are required for the creation of complex relationships.

Let the point (h, k) be the vertex of the parabola and a be the leading coefficient.

Then the equation of the parabola will be given as,

y = a(x - h)² + k

Then the parabolic function is given below.

y = (x - 2)²

The restriction of the domain will result in the inverse being a function given as,

x - 2 ≥ 0

x ≥ 2

The restriction of the domain of the function will be y = (x - 2)², x ≥ 2. Then the correct option is B.

More about the function link is given below.

https://brainly.com/question/5245372

#SPJ5

I NEED HELP WITH THIS MATH PROBLEM!!!!
Which expression is equivalent to 6f-1+7f-8?
13f+7 5f-1
-f+7 or -9+13f

Answers

Answer:

Step-by-step explanation:

6f - 1 + 7f - 8 = 6f + 7f - 1 - 8

                     = 13f - 9

Combine like terms. Like terms have same variable with same power.

Answer:

-9+13f

Step-by-step explanation:

6f+7f = 13f

-1 + -8= -9

So the sum of them is -9+13f

what is the value of the function when x = 0? ​

Answers

Answer:

f(0) = -2.

Step-by-step explanation:

This question pertains to the y-intercept. By definition, the y-intercept is the point on the graph where it crosses the y-axis, and has coordinates, (0, b). It is also the value of y when x = 0.  Looking at the graph, the point where the line crosses the y-axis is (0, -2). The y-coordinate is the y-intercept, b.

Therefore, the value of the function when x = 0 is: f(0) = -2.

Change 1/5 to a fraction

Answers

Answer:

.2

Step-by-step explanation:

Solve the following for x :
y+x=z

a. x = yz
b. x = z - y
c. x = y - z
d. x = z/y

Explain.

Answers

Answer:

B. x = z - y

Step-by-step explanation:

Move y  to the opposite side of the equals sign while remembering to make it negative:

y + x = z  

|_______|

x = z - y

Hope this helped. A brainliest would be very much appreciated. (I need 1 brainliest so I can level up) :)

Solve the following expression when
v = 10 and e= 5
2v ÷ e + 3​

Answers

Answer: 7

Step-by-step explanation:

[tex]2(10) \div 5 +3\\\\20 \div 5 + 3\\\\4+3\\\\\boxed{7}[/tex]

Illinois' population is now 12.5 million and decreasing by 0.1 million each year. On the other hand, Arizona's population is 6.9 million and increasing by 0.6 million each year. In how many years will the two states have the same population?

Answers

Answer:

8 years

Step by step guide:

100,000 x 8 = 800,000

600,000 x 8 = 4,800,000

6,900,000 + 4,800,000 = 11,700,000

12,500,000 - 800,000 = 11,700,000

Which is the graph of the following equation?
x2 = 25

Can someone please help me ASAP!

Answers

Answer:

D

Step-by-step explanation:

x^2=25

[tex]\sqrt{x^{2}[/tex] = [tex]\sqrt{25}[/tex]

x= 5

You want to buy a new app for your iPhone that costs $2.50. The tax rate is 4%. How much total do you pay for the app?

Answers

9514 1404 393

Answer:

  $2.60

Step-by-step explanation:

You pay ...

  app cost + tax

  $2.50 + 4% × $2.50 = $2.50×(1 + 4%) = 1.04×$2.50 = $2.60

You pay $2.60 for the app.

round 45,621 to the nearest 10

Answers

Answer:

45621.0

Step-by-step explanation:

Already rounded to the nearest tenth.

Answer

the answer to this question is 45620.

What is the correct number form for “seven and eighty-three thousandths?”
7.083
7.83
7.803
7.830

Answers

Answer:

7.083

Step-by-step explanation:

well there's the ones place, tenths, hundredths, and thousandths. easy math, unless i'm stoopid and this is wrong :D

7.830 is the correct answer for that question

Constance will put $500 in a savings account. Which option should she choose so that she will have the most money?
1 year at 12 percent
1 year at 9 percent
2 years at 7 percent
2 years at 5 percent

Answers

Answer:

2 years at 7 percent

Step-by-step explanation:

Let's do math on each option

$500 x .12 = $60 interest

$500 x .09 = $45 interest

$500 x 0.07 = $35 x 2 = $70 interest

$500 x 0.05 = $25 x 2 = $50 interest

Answer:

B

Step-by-step explanation:

I got 100 percent

ALGEBRA Ishiro Murakami has $15.60 deducted weekly for
family medical insurance. His employer pays 85% of the cost.
What is the annual premium?

Answers

Murakami's Annual premium which is the total value of medical insurance in a year is $5408

Percentage paid by employer = 85%Percentage paid by worker = 15%Number of weeks in a year = 52

Annual deduction from worker's salary = 52 × 15.60 = $811.2

15% of annual premium = 811.2

Let annual premium = p

Therefore,

15% × p = 811.2

0.15p = 811.2

Divide both sides by p

p = 811.20 / 0.15

p = $5408

Therefore, his annual premium is $5408

Learn more : https://brainly.com/question/18796573

Subtract 8 1/5 - 4 2/5
. Simplify the answer and write as a mixed number.

Answers

Answer:

3 4/5

Step-by-step explanation:

Determine the intercepts of the line.
Do not round your answers.
y=-2x-21y=−2x−21

Answers

Answer:

For y-intercept [ x = 0 ]

[tex]{ \rm{y = - 2x - 21}} \\ \\ { \rm{y = ( - 2 \times 0) - 21}} \\ \\ { \rm{y = - 21}} \\ \\ { \boxed{ \rm{ \: y - intercept : \: (0, \: - 21) \: }}}[/tex]

For x-intercept [ y = 0 ]

[tex]{ \rm{y = - 2x - 21}} \\ \\ { \rm{0 = - 2x - 21}} \\ \\ { \rm{2x = - 21}} \\ \\ { \rm{x = - 10.5}} \\ \\ { \boxed{ \rm{x -intercept : \: ( - 10.5, \: 0) \: }}}[/tex]

Answer:

I hope you got it right!! :)

Step-by-step explanation:

If you can clean 1/6 of your bedroom in 15 minutes, what is your rate per hour?

Answers

2/3 per hour. 15 min x 4 = 1 hour. 1/6 x 4 = 4/6. Simply 4/6 = 2/3

what value of a makes the following equation true​

Answers

Answer:

It's -30

Step-by-step explanation:

Combine like terms

 −  8  =  2 1 0

 −7  =  2 1 0

Divide both sides of the equation by the same term

− 7  =  2 1 0

-7a/-7 = 210/-7

You cancel the -7  on the left side and divide 210 by -7

and then 210/-7 is -30

Hope this helps mark me brainliest!

A coat usually sells for $62.95. It is on sale for 10% off.
What is the sale price?

Answers

The sale price is $56.66

Formula: original price * % = discount Price

Original price - discount price= sale price

bc if b = -3 1/8 and c = -4/5

Answers

Answer:

-3 37/40

Step-by-step explanation:

Other Questions
Choose all of the issues which were addressed by amendments to the US. Constitution in the World War I era.A.repeal the 18th amendmentB.the right to vote regardless of sexC.the creation of an individual income taxD.guarantee of voting rights regardless of raceE.banning of the manufacture or sale of alcoholic beverages Fill in the function table: y = -2(x - 1) please help a researcher believes that male participants will be less willing to discuss their relationship problems than female participants, and thus he makes males extra comfortable when interviewing them by nodding his head and smiling. at the end of the study, he finds that men are actually very open to talking about their relationships, even more so than women. what is the biggest problem with his findings? At a pet store the ratio of cats to dogs sold was 6 : 5. If there were 12 cats that were sold, how many dogs were sold? ....frame of that picture is broken what is 1+1-923422354x2x26757635 Please answer these 2 Which statement is TRUE? MARKING AS BRAINLIEST!! ( VALLEY FORGE, one reason why you would quit the army ? me need a lot of help Your job pays you $1600per month all taxes combined reduce your pay by 25% your current expenses are $1200 per month you want to join a health club for $75 per month can you afford it Find the value of x...... Write the equation of the line in fully simplified slope-intercept form. Determine the value of x. X 1 X 1.5A)x=1 B)x=2.5 C)x=2 D)x=2.9 what is 4-1/4 help please At a diving competition, Allison scored 78.5 on her first dive. Hannah's score on her first dive was 74.65. How many more points did Allison score on her first dive than Hannah? Show your work. what is de las casas trying to say about divine providence and the establishment of the new world? sin^2 x - 2sinx + 2 = sin^2 3x 1. What is abstraction? Describe it in your OWN words. if a man has no common experience in communication do you think their will be an underatanding among people in the world?